-
Notifications
You must be signed in to change notification settings - Fork 1
/
Copy pathPOLY2007.tex
1517 lines (976 loc) · 72.4 KB
/
POLY2007.tex
1
2
3
4
5
6
7
8
9
10
11
12
13
14
15
16
17
18
19
20
21
22
23
24
25
26
27
28
29
30
31
32
33
34
35
36
37
38
39
40
41
42
43
44
45
46
47
48
49
50
51
52
53
54
55
56
57
58
59
60
61
62
63
64
65
66
67
68
69
70
71
72
73
74
75
76
77
78
79
80
81
82
83
84
85
86
87
88
89
90
91
92
93
94
95
96
97
98
99
100
101
102
103
104
105
106
107
108
109
110
111
112
113
114
115
116
117
118
119
120
121
122
123
124
125
126
127
128
129
130
131
132
133
134
135
136
137
138
139
140
141
142
143
144
145
146
147
148
149
150
151
152
153
154
155
156
157
158
159
160
161
162
163
164
165
166
167
168
169
170
171
172
173
174
175
176
177
178
179
180
181
182
183
184
185
186
187
188
189
190
191
192
193
194
195
196
197
198
199
200
201
202
203
204
205
206
207
208
209
210
211
212
213
214
215
216
217
218
219
220
221
222
223
224
225
226
227
228
229
230
231
232
233
234
235
236
237
238
239
240
241
242
243
244
245
246
247
248
249
250
251
252
253
254
255
256
257
258
259
260
261
262
263
264
265
266
267
268
269
270
271
272
273
274
275
276
277
278
279
280
281
282
283
284
285
286
287
288
289
290
291
292
293
294
295
296
297
298
299
300
301
302
303
304
305
306
307
308
309
310
311
312
313
314
315
316
317
318
319
320
321
322
323
324
325
326
327
328
329
330
331
332
333
334
335
336
337
338
339
340
341
342
343
344
345
346
347
348
349
350
351
352
353
354
355
356
357
358
359
360
361
362
363
364
365
366
367
368
369
370
371
372
373
374
375
376
377
378
379
380
381
382
383
384
385
386
387
388
389
390
391
392
393
394
395
396
397
398
399
400
401
402
403
404
405
406
407
408
409
410
411
412
413
414
415
416
417
418
419
420
421
422
423
424
425
426
427
428
429
430
431
432
433
434
435
436
437
438
439
440
441
442
443
444
445
446
447
448
449
450
451
452
453
454
455
456
457
458
459
460
461
462
463
464
465
466
467
468
469
470
471
472
473
474
475
476
477
478
479
480
481
482
483
484
485
486
487
488
489
490
491
492
493
494
495
496
497
498
499
500
501
502
503
504
505
506
507
508
509
510
511
512
513
514
515
516
517
518
519
520
521
522
523
524
525
526
527
528
529
530
531
532
533
534
535
536
537
538
539
540
541
542
543
544
545
546
547
548
549
550
551
552
553
554
555
556
557
558
559
560
561
562
563
564
565
566
567
568
569
570
571
572
573
574
575
576
577
578
579
580
581
582
583
584
585
586
587
588
589
590
591
592
593
594
595
596
597
598
599
600
601
602
603
604
605
606
607
608
609
610
611
612
613
614
615
616
617
618
619
620
621
622
623
624
625
626
627
628
629
630
631
632
633
634
635
636
637
638
639
640
641
642
643
644
645
646
647
648
649
650
651
652
653
654
655
656
657
658
659
660
661
662
663
664
665
666
667
668
669
670
671
672
673
674
675
676
677
678
679
680
681
682
683
684
685
686
687
688
689
690
691
692
693
694
695
696
697
698
699
700
701
702
703
704
705
706
707
708
709
710
711
712
713
714
715
716
717
718
719
720
721
722
723
724
725
726
727
728
729
730
731
732
733
734
735
736
737
738
739
740
741
742
743
744
745
746
747
748
749
750
751
752
753
754
755
756
757
758
759
760
761
762
763
764
765
766
767
768
769
770
771
772
773
774
775
776
777
778
779
780
781
782
783
784
785
786
787
788
789
790
791
792
793
794
795
796
797
798
799
800
801
802
803
804
805
806
807
808
809
810
811
812
813
814
815
816
817
818
819
820
821
822
823
824
825
826
827
828
829
830
831
832
833
834
835
836
837
838
839
840
841
842
843
844
845
846
847
848
849
850
851
852
853
854
855
856
857
858
859
860
861
862
863
864
865
866
867
868
869
870
871
872
873
874
875
876
877
878
879
880
881
882
883
884
885
886
887
888
889
890
891
892
893
894
895
896
897
898
899
900
901
902
903
904
905
906
907
908
909
910
911
912
913
914
915
916
917
918
919
920
921
922
923
924
925
926
927
928
929
930
931
932
933
934
935
936
937
938
939
940
941
942
943
944
945
946
947
948
949
950
951
952
953
954
955
956
957
958
959
960
961
962
963
964
965
966
967
968
969
970
971
972
973
974
975
976
977
978
979
980
981
982
983
984
985
986
987
988
989
990
991
992
993
994
995
996
997
998
999
1000
-------------------------------------------------------------------------------
\begin{problem}[Posted by \href{https://artofproblemsolving.com/community/user/1838}{paladin8}]
$P(x)$ is a polynomial of degree $3n$ such that
\begin{eqnarray*} P(0) = P(3) = \cdots &=& P(3n) = 2, \\ P(1) = P(4) = \cdots &=& P(3n-2) = 1, \\ P(2) = P(5) = \cdots &=& P(3n-1) = 0, \quad\text{ and }\\ && P(3n+1) = 730.\end{eqnarray*}
Determine $n$.
\flushright \href{https://artofproblemsolving.com/community/c6h62982}{(Link to AoPS)}
\end{problem}
\begin{solution}[by \href{https://artofproblemsolving.com/community/user/14052}{t0rajir0u}]
Eek... but if I may offer a possible simplification of the problem: Consider $Q(x) = P(x) - 1$. Then the problem can be rewritten
$Q(3k) = 1, k \le n$
$Q(3k-2) = 0, k \le n$
$Q(3k-1) = -1, k \le n$
$Q(3n+1) = 729 = 3^6 = 3 \times 3^2 \times 3^3$
Hope that leads someone to a solution 'cause I'm stuck and I have homework to do :) (Personally, I think $n = 3$ or $n = 6$)
\end{solution}
\begin{solution}[by \href{https://artofproblemsolving.com/community/user/13833}{Raluca}]
We have
$P(x)=-x+2+3[x\/3], \ \forall x\in\{0,1,2,...,3n\}$,
so
$\frac{P(x)+x-2}{3}=[x\/3], \ \forall x\in\{0,1,2,...,3n\}$.
Denote
$F(x)=\frac{P(x)+x-2}{3}$
Since $P(3n+1)=730$, we get $F(3n+1)=n+243$.
We havo to find $n$ such that polynomial $F$ should satisfy:
$(i) \ F(x)=[x\/3], \ \forall x\in\{0,1,2,...,3n\}$;
$(ii) \ F(3n+1)=n+243$;
$(iii) \deg(F)=3n$.
I don't know if it could be helpful :? .
\end{solution}
\begin{solution}[by \href{https://artofproblemsolving.com/community/user/6912}{chess64}]
Does anyone have a solution??
\end{solution}
\begin{solution}[by \href{https://artofproblemsolving.com/community/user/6741}{jin}]
\begin{tcolorbox}Eek... but if I may offer a possible simplification of the problem: Consider $Q(x) = P(x) - 1$. Then the problem can be rewritten
$Q(3k) = 1, k \le n$
$Q(3k-2) = 0, k \le n$
$Q(3k-1) = -1, k \le n$
Hope that leads someone to a solution 'cause I'm stuck and I have homework to do :) (Personally, I think $n = 3$ or $n = 6$)\end{tcolorbox}
So we can get $f(3n+1)=(-1)^n\frac{2}{3}(\sqrt{3})^{3n+2}\cos\frac{3n+2}{6}\pi$ by Langrange Interpolation.
So $n=4$.
\end{solution}
\begin{solution}[by \href{https://artofproblemsolving.com/community/user/10301}{maokid7}]
could you please explain some more.
\end{solution}
\begin{solution}[by \href{https://artofproblemsolving.com/community/user/6551}{perfect_radio}]
Using Lagrange Interpolation I get that $P(3n+1) = \sum_{k=0}^{3n}(-1)^{3n-k}\binom{3n+1}{k}f(k)$.
Now you could use the old trick with the roots of unity, but I don't see any way of keeping the computations short.
\end{solution}
\begin{solution}[by \href{https://artofproblemsolving.com/community/user/20099}{pardesi}]
Before wriring down the proof(asuuming it to be correct) an intutive note since i was doing complex no.s these days period three reminds me of $\omega$
\begin{italicized}Proof\end{italicized}
a bit guess work and manipulation may lead u to conclude that
$P(k)-1$ $=$ $\frac{2Ime^{\frac{\pi i}{3}}\omega^{k}}{\sqrt 3}$ for the given values of $k$
claim is
$P(x)=1+\frac{2\sum_{k=0}^{3n}\binom{x}{k}Ime^{\frac{pi i}{3}}(\omega-1)^{k}}{\sqrt 3}$
clearly $P(j)=1+\frac{2Ime^{\frac{\pi i}{3}}\omega^{k}}{\sqrt 3}$ for $j=0,1,2\dots 3n$
so the polynomial cannot be anything but the above one
a bit of manipulation leads to
$P(3n+1)=\frac{-2 Ime^{\frac{\pi i}{3}}(\omega-1)^{k}}{\sqrt 3}$
note $\omega-1=1\sqrt 3 e^{\frac{\pi i}{3}}$
we get by De-moivre's Theorem $P(3n+1)=730=3^{6}+1$
this gives $n=4$
\end{solution}
\begin{solution}[by \href{https://artofproblemsolving.com/community/user/29428}{pco}]
\begin{tcolorbox}could you please explain some more.\end{tcolorbox}
1) For integer $k\in[0,3n]$, the unique polynomial $P_{k}$ of degree $3n$ and such that $P_{k}(k)=1$ and $P_{k}(p)=0$ for any integer integer $p\neq k\in[0,3n]$ is :
$P_{k}(x)=\frac{1}{(-1)^{3n-k}k!(3n-k)!}\prod_{i=0,i\neq k}^{i=3n}(x-i)$. Notice that $P_{k}(3n+1)=(-1)^{3n-k}\binom{3n+1}{k}$
2) So the requested polynomial is well defined by its values for 0 to $3n$ and we have :
$P(x)=\sum_{k=0}^{k=n}2P_{3k}(x)+\sum_{k=0}^{k=n-1}P_{3k+1}(x)$
3) We can now compute $S=P(3n+1)$
$S=\sum_{k=0}^{k=n}2(-1)^{3n-3k}\binom{3n+1}{3k}+\sum_{k=0}^{k=n-1}(-1)^{3n-3k-1}\binom{3n+1}{3k+1}$
Let $A=\sum_{k=0}^{k=n}(-1)^{3k}\binom{3n+1}{3k}$, $B=\sum_{k=0}^{k=n}(-1)^{3k+1}\binom{3n+1}{3k+1}$ and $C=\sum_{k=0}^{k=n-1}(-1)^{3k+2}\binom{3n+1}{3k+2}$. We have :
$S=(-1)^{3n}2A+(-1)^{3n}B+1$
$(1-1)^{3n+1}=0=A+B+C$
$(1-j)^{3n+1}=A+Bj+Cj^{2}$ where $j$ is one one of the complex root of $x^{3}=1$.
$(1-j^{2})^{3n+1}=A+Bj^{2}+Cj$
And : $A=\frac{(1-j)^{3n+1}+(1-j^{2})^{3n+1}}{3}$ and $B=\frac{j^{2}(1-j)^{3n+1}+j(1-j^{2})^{3n+1}}{3}$
$S=(-1)^{3n}2A+(-1)^{3n}B+1=1-j(j-1)^{3n}-j^{2}(j^{2}-1)^{3n}$
If $n$ is odd ($n=2p+1$), we have $S=1+(-1)^{p}3^{3p+2}$
Id $n$ is even ($n=2p$), we have $S=1+(-1)^{p}3^{3p}$
4) In our case, we need to solve $S=730=3^{6}+1$
Clearly $p=2$ and $n=4$
--
Patrick
\end{solution}
\begin{solution}[by \href{https://artofproblemsolving.com/community/user/29176}{singapore1728}]
hey, there is a solution using difference operator by Dr Kin Yi Li in his journal "Mathematical Excalibur", which I think is more succinct than lagrange interpolation. here is the link:
http://www.math.ust.hk\/excalibur\/v11_n5.pdf
\end{solution}
*******************************************************************************
-------------------------------------------------------------------------------
\begin{problem}[Posted by \href{https://artofproblemsolving.com/community/user/5820}{N.T.TUAN}]
Unitary quadratic trinomials $ f(x)$ and $ g(x)$ satisfy the following interesting condition: $ f(g(x)) = 0$ and $ g(f(x)) = 0$ do not have real roots. Prove that at least one of equations $ f(f(x)) = 0$ and $ g(g(x)) = 0$ does not have real roots too.
\begin{italicized}S. Berlov \end{italicized}
\flushright \href{https://artofproblemsolving.com/community/c6h147156}{(Link to AoPS)}
\end{problem}
\begin{solution}[by \href{https://artofproblemsolving.com/community/user/29428}{pco}]
Hello,
\begin{tcolorbox}\begin{italicized}S. Berlov \end{italicized}
Unitary quadratic trinomials $f(x)$ and $g(x)$ satisfy
the following interesting condition: $f(g(x))=0$ and $g(f(x))=0$
do not have real roots. prove that at least one of equations
$f(f(x))=0$ and $g(g(x))=0$ does not have roots too.\end{tcolorbox}
Assume $f(f(x))=0$ and $g(g(x))=0$ have roots. Then $f(x)$ have roots too, say $x_{1}$ and $x_{2}$ with $x_{1}\leq x_{2}$ and $g(x)$ have roots too, say $y_{1}$ and $y_{2}$ with $y_{1}\leq y_{2}$
We can assume $x_{2}\leq y_{2}$ (else we just have to swap f and g)
Since $f(f(x))=0$ has root $x_{3}$, $f(x_{3})=x_{1}$ or $f(x_{3})=x_{2}$, so $f(x_{3})\leq y_{2}$
But $f(x_{3})\leq y_{2}$ and $f(x) \rightarrow+\infty$ when $x \rightarrow+\infty$ implies it exists $x_{4}$ such that $f(x_{4})=y_{2}$ $\Rightarrow $ $g(f(x_{4}))=g(y_{2})=0$
So, if $f(f(x))=0$ and $g(g(x))=0$ has roots, then either $f(g(x))=0$ or $g(f(x))=0$ has root.
So, if neither $f(g(x))=0$ nor $g(f(x))=0$ has root, either $f(f(x))=0$ or $g(g(x))=0$ don't have roots.
Q.E.D.
--
Patrick
\end{solution}
\begin{solution}[by \href{https://artofproblemsolving.com/community/user/34380}{math10}]
\begin{tcolorbox}\begin{italicized}S. Berlov \end{italicized}
Unitary quadratic trinomials $ f(x)$ and $ g(x)$ satisfy
the following interesting condition: $ f(g(x)) = 0$ and $ g(f(x)) = 0$
do not have real roots. prove that at least one of equations
$ f(f(x)) = 0$ and $ g(g(x)) = 0$ does not have roots too.\end{tcolorbox}
Easy problem. Here my solution .
If equation $ f(x).g(x) = 0$ haven't root then equation $ f(f(x) = 0$ and $ g(g(x)) = 0$ haven't root.
Let $ x_1,x_2$ be root of equation $ f(x) = 0$,$ x_3,x_4$ be root of equation $ g(x) = 0$
Suppose 2 equation $ f(f(x)) = 0$ and $ g(g(x)) = 0$ have root.
So $ \triangle_f - 4x_1\geq 0$; $ \triangle_f - 4x_2\geq 0$; $ \triangle_g - 4x_4\geq 0$; $ \triangle_g - 4x_3\geq 0$
So $ 2\triangle_f + 2\triangle_g - 4\sum_{i = 1}^{4}x_i\geq 0$
From equation $ f(g(x)) = 0$ haven't root we have equation $ g(x) = x_1$ haven't root .So $ \triangle_g - 4x_1 < 0$
Similary we have $ \triangle_g - 4x_2 < 0$; $ \triangle_f - 4x_3 < 0$ ; $ \triangle_f - 4x_4 < 0$
So $ 2\triangle_f + 2\triangle_g - 4\sum_{i = 1}^{4}x_i < 0$ QED.
\end{solution}
*******************************************************************************
-------------------------------------------------------------------------------
\begin{problem}[Posted by \href{https://artofproblemsolving.com/community/user/5820}{N.T.TUAN}]
Do there exist non-zero reals $a$, $b$, $c$ such that, for any $n>3$, there exists a polynomial $P_{n}(x) = x^{n}+\dots+a x^{2}+bx+c$, which has exactly $n$ (not necessary distinct) integral roots?
\begin{italicized}N. Agakhanov, I. Bogdanov\end{italicized}
\flushright \href{https://artofproblemsolving.com/community/c6h147176}{(Link to AoPS)}
\end{problem}
\begin{solution}[by \href{https://artofproblemsolving.com/community/user/29428}{pco}]
The answer is no.
Let $r_{i}$ be the roots of $P_{n}(x)$. We have $P_{n}(x)=\prod_{i=1}^{n}(x-r_{i})$. Then :
$(-1)^{n}\prod_{i=1}^{n}r_{i}=c$
$(-1)^{n-1}(\prod_{i=1}^{n}r_{i})(\sum_{i=1}^{n}\frac{1}{r_{i}})=b$ (no root is zero since c is non-zero)
For easier view, let $x_{i}=-\frac{1}{r_{i}}$. We have then : $\prod_{i=1}^{n}x_{i}=\frac{1}{c}$ and $\sum_{i=1}^{n}x_{i}=\frac{b}{c}$
Then, let $G$ be the geometric mean of $x_{i}$. We have $G=(\frac{1}{c})^{\frac{1}{n}}$
Let A be the arithmetic mean of $x_{i}$. We have $A=\frac{b}{n*c^{}}$
We know that $G\leq A$, so $(\frac{1}{c})^{\frac{1}{n}}\leq \frac{b}{n*c^{}}$ $\forall n>3$
And this is impossible since LHS has a limit 1 when n grows while RHS decreases towards 0 when n grows.
--
Patrick
\end{solution}
\begin{solution}[by \href{https://artofproblemsolving.com/community/user/16261}{Rust}]
\begin{tcolorbox}We know that $G\leq A$, so $(\frac{1}{c})^{\frac{1}{n}}\leq \frac{b}{n*c^{}}$ $\forall n>3$
And this is impossible since LHS has a limit 1 when n grows while RHS decreases towards 0 when n grows.
--
Patrick\end{tcolorbox}
Let $x_{1}=x_{2}=1,x_{3}=x_{4}=-1$. $A=0<G=1$. :D
It is possible.
\end{solution}
\begin{solution}[by \href{https://artofproblemsolving.com/community/user/29428}{pco}]
\begin{tcolorbox} Let $x_{1}=x_{2}=1,x_{3}=x_{4}=-1$. $A=0<G=1$. :D
It is possible.\end{tcolorbox}
Yes, You're right, $G\leq A$ is true if all $x_{i}$ are $\geq 0$. :blush:
So here is the correction :
The answer is no.
Let $r_{i}$ be the roots of $P_{n}(x)$. We have $P_{n}(x)=\prod_{i=1}^{n}(x-r_{i})$. Then :
$(-1)^{n}\prod_{i=1}^{n}r_{i}=c$
$(-1)^{n-1}(\prod_{i=1}^{n}r_{i})(\sum_{i=1}^{n}\frac{1}{r_{i}})=b$ (no root is zero since c is non-zero)
$(-1)^{n-2}(\prod_{i=1}^{n}r_{i})(\sum_{i\neq j}\frac{1}{r_{i}r_{j}})=(-1)^{n-2}(\prod_{i=1}^{n}r_{i})\frac{1}{2}( (\sum_{i=1}^{n}\frac{1}{r_{i}})^{2}-(\sum_{i=1}^{n}\frac{1}{r_{i}^{2}}))=a$
And so :
$\prod_{i=1}^{n}r_{i}=(-1)^{n}c$, $\sum_{i=1}^{n}\frac{1}{r_{i}}=-\frac{b}{c}$ and $\sum_{i=1}^{n}\frac{1}{r_{i}^{2}}=\frac{b^{2}}{c^{2}}-2\frac{a}{c}$
For easier view, let $x_{i}=-\frac{1}{r_{i}}$. We have then :
$\prod_{i=1}^{n}x_{i}=\frac{1}{c}$, $\sum_{i=1}^{n}x_{i}=\frac{b}{c}$ and $\sum_{i=1}^{n}x_{i}^{2}=\frac{b^{2}}{c^{2}}-2\frac{a}{c}=u^{2}$ for a certain real $u\geq 0$ (and the constraint $b\geq 2ac$)
Let then G be the geometric mean of $|x_{i}|$. We have $G=(\frac{1}{|c|})^{\frac{1}{n}}$.
Let then Q be the quadratic mean of $|x_{i}|$. We have $Q=\frac{u}{n^{\frac{1}{2}}}$
We know that, for a given set of positive (or zero) $|x_{i}|, G\leq Q$. So, $(\frac{1}{|c|})^{\frac{1}{n}}\leq \frac{u}{n^{\frac{1}{2}}}$ $\forall n>3$
And this is impossible since LHS have limit 1 when n increases while RHS have limit 0.
--
Patrick
\end{solution}
\begin{solution}[by \href{https://artofproblemsolving.com/community/user/16261}{Rust}]
Yes. You are right.
\begin{tcolorbox} For easier view, let $x_{i}=-\frac{1}{r_{i}}$. We have then :
$\prod_{i=1}^{n}x_{i}=\frac{1}{c}$, $\sum_{i=1}^{n}x_{i}=\frac{b}{c}$ and $\sum_{i=1}^{n}x_{i}^{2}=\frac{b^{2}}{c^{2}}-2\frac{a}{c}=u^{2}$ for a certain real $u\geq 0$ (and the constraint $b\geq 2ac$)
Patrick\end{tcolorbox}
But $b^{2}\ge 4ac .$
\end{solution}
\begin{solution}[by \href{https://artofproblemsolving.com/community/user/11003}{pavel kozlov}]
From $\prod_{i=1}^{n}{r_{i}}^{2}=c^{2}$ and $\sum_{i=1}^{n}\frac{1}{{r_{i}}^{2}}=\frac{b^{2}}{c^{2}}-2\frac{a}{c}$ follows that their product (which is equal to $b^{2}-2ac$) is limited. On the other hand this product is the sum of $n$ positive numbers of a form $\frac{\prod_{i=1}^{n}{r_{i}}^{2}}{{r_{i}}^{2}}$. Hence $n\leq b^{2}-2ac$ :o :idea: It is impossible!
\end{solution}
*******************************************************************************
-------------------------------------------------------------------------------
\begin{problem}[Posted by \href{https://artofproblemsolving.com/community/user/7956}{pureblack2003}]
Find all polynomials $P \in \mathbb{R}[X]$ such that
\[(x-4)P(x+1)-xP(x)+20=0,\]
for all $x \in \mathbb R$.
\flushright \href{https://artofproblemsolving.com/community/c6h149112}{(Link to AoPS)}
\end{problem}
\begin{solution}[by \href{https://artofproblemsolving.com/community/user/5820}{N.T.TUAN}]
[hide="A hint"]Because $20=5x-5(x-4)$ we have $(x-4)P(x+1)-xP(x)+5x-5(x-4)=0$ or $(x-4)(P(x+1)-5)=x(P(x)-5)$, then put $Q(x)=P(x)-5$ we have $(x-4)Q(x+1)=xQ(x)$. Now it is well know![\/hide]
\end{solution}
\begin{solution}[by \href{https://artofproblemsolving.com/community/user/11776}{mathisfun1}]
It's straightforward to show via induction that $P(x) = 5$ for all positive integers. Hence $P(x)=5$.
\end{solution}
\begin{solution}[by \href{https://artofproblemsolving.com/community/user/5820}{N.T.TUAN}]
:blush: Ok! But i think your method not works with following:
Let $a,b\in\mathbb{R}$, find all $P\in\mathbb{R}[x]$ such that \[xP(x-a)\equiv (x-b)P(x).\]
Happy hunting!
\end{solution}
\begin{solution}[by \href{https://artofproblemsolving.com/community/user/5820}{N.T.TUAN}]
\begin{tcolorbox}It's straightforward to show via induction that $P(x) = 5$ for all positive integers. Hence $P(x)=5$.\end{tcolorbox}
Sorry, But you can't have $P(5)=5$ :P
\end{solution}
\begin{solution}[by \href{https://artofproblemsolving.com/community/user/11776}{mathisfun1}]
Since $P(4) = 5$, $4P(5)-20-4P(5)+20=0$, so we can certainly make $P(5)=5$. But I think you mean that $P(5)$ does not have to equal 5. The problem arises in my induction: If $P(n) = 5$, then $nP(n+1)-5n-4P(n+1)+20 = 0 \Rightarrow P(n+1) = \frac{5(n-4)}{n-4}= 5$. This breaks down when $n=4$. If $P(5) \ne 5$, then let $P(x) = a_{n}x^{n}+a_{n-1}x^{n-1}+...+a_{1}x+a_{0}$. Comparing the leading terms of $x(P(x+1)-P(x))$ and $4P(x+1)$, we find that $n=4$. So $P(x) = a_{4}x^{4}+a_{3}x^{3}+a_{2}x^{2}+a_{1}x+a_{0}$. $P(1)=P(2)=P(3)=P(4)=5$, etc.
\end{solution}
\begin{solution}[by \href{https://artofproblemsolving.com/community/user/5820}{N.T.TUAN}]
Final, what is your answer?
\end{solution}
\begin{solution}[by \href{https://artofproblemsolving.com/community/user/4229}{scorpius119}]
[hide="following the hint"]
From $(x-4)Q(x+1)=xQ(x)$, we immediately see that 1 and 4 are roots by plugging in $x=0,4$. Then plug in $x=1,3$ to find that 2 and 3 are also roots. Therefore, there exists a polynomial $C(x)$ such that
\[Q(x)=(x-1)(x-2)(x-3)(x-4)C(x)\]
and the equation becomes $C(x+1)=C(x)$. This means $C$ is constant (since $C(x)-C(0)$ has roots at all nonnegative integers $x$, so has infinitely many roots and is therefore the zero polynomial). So we obtain the solution
\[P(x)=5+k(x-1)(x-2)(x-3)(x-4)\]
for any constant $k$.
[\/hide]
\end{solution}
\begin{solution}[by \href{https://artofproblemsolving.com/community/user/11776}{mathisfun1}]
\begin{tcolorbox}Final, what is your answer?\end{tcolorbox}
Using Scorpius119's ending: $P(1) =P(2)=P(3)=P(4) = 5 \Rightarrow P(x)-5 = a_{4}x^{4}+a_{3}x^{3}+a_{2}x^{2}+a_{1}x_{1}+a_{0}-5 = k(x-1)(x-2)(x-3)(x-4)$.
:blush: To be honest, I didn't see the more efficient ending -- I was thinking about substituting back into the original equation and getting a system of equations.
\end{solution}
\begin{solution}[by \href{https://artofproblemsolving.com/community/user/5820}{N.T.TUAN}]
Ok, Now we can continue for :
\begin{tcolorbox}:blush: Ok! But i think your method not works with following:
Let $a,b\in\mathbb{R}$, find all $P\in\mathbb{R}[x]$ such that
\[xP(x-a)\equiv (x-b)P(x). \]
Happy hunting! \end{tcolorbox}
\end{solution}
\begin{solution}[by \href{https://artofproblemsolving.com/community/user/29428}{pco}]
\begin{tcolorbox}Ok, Now we can continue for :
Let $a,b\in\mathbb{R}$, find all $P\in\mathbb{R}[x]$ such that
\[xP(x-a)\equiv (x-b)P(x). \]
\end{tcolorbox}
1) if $a=0$ and $b=0$, any $P(x)$ is solution
2) if $a=0$ and $b\neq 0$, $P(x)=0$ is the only solution
3) if $a\neq 0$ and $b=0$, $P(x-a)=P(x)$ $\forall x\neq 0$ $\Rightarrow $ $P(x)=c$
4) if $ab\neq 0$ :
$x=0$ in $xP(x-a)=(x-b)P(x)$ $\Rightarrow $ $P(0)=0$
If $P(r)=0$ with $r\neq 0$, then $P(r-a)=0$ $\Rightarrow $ all roots of P are $0,a, 2a, \cdot, ua$ for some u of $\mathbb{Z}^{+}$
$x=b$ in $xP(x-a)=(x-b)P(x)$ $\Rightarrow $ $P(b-a)=0$
If $P(r)=0$ with $r\neq b-a$, then $P(r+a)=0$ $\Rightarrow $ all roots of P are $b-a,b-2a, b-3a, ..., b-ka$ for some k of $\mathbb{N}$
Hence we must have $b=ka$ and all roots are $0, a, 2a, ..., (k-1)a$ $\Rightarrow $ $P(x)=c\prod_{i=0}^{k-1}(x-ia)^{n_{i}}$
Identification in initial equation gives $n_{i}=1$
As a result, we have :
a) if $a=0$ and $b=0$, then $P(x)$ is any
b) if $a=0$ and $b\neq 0$, then $P(x)\equiv 0$
c) if $a\neq 0$ and $b=0$, then $P(x)\equiv c$
d) if $ab\neq 0$ then it must exist k in $\mathbb{N}$ such that $b=ka$ and solution is $P(x)=c\prod_{i=0}^{k-1}(x-ia)$
--
Patrick
\end{solution}
\begin{solution}[by \href{https://artofproblemsolving.com/community/user/18420}{aviateurpilot}]
$E=\{k\in \mathbb{C}| \ p(k)=0\}$
$a=b=0$ => $S=R[X]$
$a\neq 0,b=0$ => $P\equiv c$.
if $ab\neq 0$
if $|E-a\mathbb{N}|\neq 0$ such that $P(k)=0$
then $\forall (s,k)\in \mathbb{N}\times (E-a\mathbb{N}),$ $p(k-sa)=0$ so, $p\equiv 0$
if $p\not\equiv 0$
$E\subseteq a\mathbb{N}$
we can suppose that $am=MAX(E)$ so, it's evident that $E=\{am,a(m-1),a(m-2),..,0\}$.
then $p(x)=c\prod_{i=0}^{m}(x-ia)^{n_{i}},\ c\in\mathbb{R}^{*}$
and $x\prod_{i=1}^{m+1}(x-ia)^{n_{i-1}}=(x-b)\prod_{i=0}^{m}(x-ia)^{n_{i}}$
so $x-b=x-(m+1)a\ (b\ must\ be\ in\ a\mathbb{N}^{*})$( and $n_{i}=n={i-1}$ for $i=1..m$
finally.
$p(x)=cx\prod_{i=1}^{m}(x-ia)^{n}\ ,with\ (c,m,n)\in \mathbb{R}\times \mathbb{N}^{*}\times \mathbb{N}$
\end{solution}
*******************************************************************************
-------------------------------------------------------------------------------
\begin{problem}[Posted by \href{https://artofproblemsolving.com/community/user/8143}{Jutaro}]
Let $f(x)$ be a cubic polynomial with rational coefficients. If the graph of $f(x)$ is tangent to the $x$ axis, prove that the roots of $f(x)$ are all rational.
\flushright \href{https://artofproblemsolving.com/community/c6h149390}{(Link to AoPS)}
\end{problem}
\begin{solution}[by \href{https://artofproblemsolving.com/community/user/29428}{pco}]
\begin{tcolorbox}Let $f(x)$ be a cubic polynomial with rational coefficients. If the graph of $f(x)$ is tangent to the $x$ axis, prove that the roots of $f(x)$ are all rational.\end{tcolorbox}
If $x=r$ is the point where the graph of $f(x)$ is tangent to the $x$ axis, then $f(r)=f'(r)=0$ and r is at least a double root.
Polynomial division of $f(x)$ by $f'(x)$ gives $f(x)=(ax+b)f'(x)+px+q$, where $(a,b,p,q)$ are all rationals.
So we have $pr+q=0$ and r is rational.
Then polynomial division of $f(x)$ by $(x-r)^{2}$ gives $f(x)=(ux+v)(x-r)^{2}$, with $(u,v)$ rational and hence the third root is rational too.
--
Patrick
\end{solution}
\begin{solution}[by \href{https://artofproblemsolving.com/community/user/58773}{vijaymenon}]
actually i do not understand the solution .please help me giving a detailed explanation and solution.
please help,
\end{solution}
\begin{solution}[by \href{https://artofproblemsolving.com/community/user/58773}{vijaymenon}]
please someone reply
\end{solution}
\begin{solution}[by \href{https://artofproblemsolving.com/community/user/47249}{Makoto Kohno}]
Hence all coefficients of $ f(x)$ are rationals, all coefficients of $ f'(x)$ are rationals.
So
\begin{tcolorbox}
Polynomial division of $ f(x)$ by $ f'(x)$ gives $ f(x) = (ax + b)f'(x) + px + q$, where $ (a,b,p,q)$ are all rationals.
\end{tcolorbox}
And hence $ f(r)=f'(r)=0$ ,
\begin{tcolorbox}
So we have $ pr + q = 0$ and r is rational.
\end{tcolorbox}
And then all coefficients of $ (x-r)^{2}$ are rationals, so
\begin{tcolorbox}
Then polynomial division of $ f(x)$ by $ (x - r)^{2}$ gives $ f(x) = (ux + v)(x - r)^{2}$, with $ (u,v)$ rational and hence the third root is rational too.
\end{tcolorbox}
\end{solution}
\begin{solution}[by \href{https://artofproblemsolving.com/community/user/100136}{DavyBa}]
I withdraw my comment.
\end{solution}
\begin{solution}[by \href{https://artofproblemsolving.com/community/user/212803}{JashBlued}]
\begin{tcolorbox}The statement of this problem is incorrect. If f(x) = x^3 + 2x, then its three roots are: 0, i√2, and -i√2 which are not all rational. Its graph is tangent to the X axis in x = 0. The derivative f`(0) is not equal to 0, but the second derivative f``(0) is. The statement probably should be: "all its real roots are rational".\end{tcolorbox}
Does the polynomial f(x) = x^3 + 3x have 0 as a root of multiplicity 2? Not... right?...
\end{solution}
\begin{solution}[by \href{https://artofproblemsolving.com/community/user/155347}{Wave-Particle}]
No, it does not have a root of 0 multiplicity 2
\end{solution}
\begin{solution}[by \href{https://artofproblemsolving.com/community/user/146669}{trumpeter}]
\begin{tcolorbox}The statement of this problem is incorrect. If f(x) = x^3 + 2x, then its three roots are: 0, i√2, and -i√2 which are not all rational. Its graph is tangent to the X axis in x = 0. The derivative f`(0) is not equal to 0, but the second derivative f``(0) is. The statement probably should be: "all its real roots are rational".\end{tcolorbox}
Actually, the graph is not tangent to the $x$-axis at $x=0$. This is called an "inflection point" or "saddle point", where it sort of curves out. These are identified when $f''(x)=0$.
To solve the original problem, calculus actually is not required.
[hide=Solution]
It is well known that a polynomial is tangent to the $x$-axis at $x=r$ when the multiplicity of $r$ in the polynomial is a positive even integer.
Thus, the multiplicity must be $2$. Let $f(x)=a_3x^3+a_2x^2+a_1x+a_0$. Then, the sum of the roots (counting multiplicity) is $-\frac{a_2}{a_3}$, or a rational number. Thus, the final root is just $-\frac{a_2}{a_3}-2r$, which is a rational number. Thus, all three roots of $f(x)$ are rational.
Q.E.D.
[\/hide]
\end{solution}
\begin{solution}[by \href{https://artofproblemsolving.com/community/user/100136}{DavyBa}]
Yes, you are correct. I realized it myself just recently. The graph of this polynomial is not tangent to X axis since the first derivative at x = 0 is positive. It was a temporary blackout. :)
Thank you for your reply.
Also, thank you for posting the solution. May I ask you about one detail in this solution for my understanding?
Since the multiplicity of r in such special cubic polynomial is 2, it's clear that such cubic polynomial has 3 real roots: r, r, and some real root q. It's also clear that the sum 2r+q (and the product (r^2)q, etc.) of these three real roots are rational (Vieta's formulas). How does it prove that all three roots are rational?
\end{solution}
\begin{solution}[by \href{https://artofproblemsolving.com/community/user/100136}{DavyBa}]
It's clear how to prove it by using calculus. Let's examine two types of cubic polynomials whose graphs are tangent to X axis. Firstly, consider polynomials f(x) = a(x-b)^3 with rational a and b, that have root r = b with multiplicity 3. Both first and second derivatives f`(b) = f``(b) = 0, so its graph is tangent to X axis and b is the inflection point. All three roots are r = b (which is a rational number).
The second case is when root r of polynomial ax^3 + bx^2 + cx + d has multiplicity 2. Then, r is a local extremum. In this case, f(r) = f`(r) = 0. Polynomial division f(x)\/f`(x) gives us the remainder px + q in which p and q are rational numbers. From f(r) = f`(r) = 0 it follows that pr + q = 0 and that r is a rational number. From this it follows by Vieta's formula that the third root is also rational. Q.E.D.
\end{solution}
*******************************************************************************
-------------------------------------------------------------------------------
\begin{problem}[Posted by \href{https://artofproblemsolving.com/community/user/16383}{M4RI0}]
Find all polynomials $f(x)$ with real coefficients satisfying
\[f(x^{2})=f(x)f(x-1) \quad \forall x \in \mathbb R.\]
\flushright \href{https://artofproblemsolving.com/community/c6h151076}{(Link to AoPS)}
\end{problem}
\begin{solution}[by \href{https://artofproblemsolving.com/community/user/29428}{pco}]
\begin{tcolorbox}Find all real polynomials $f(x)$ satisfying
$f(x^{2})=f(x)f(x-1)$ for all $x$\end{tcolorbox}
Nice question.
Let first consider that f is a complex polynomial.
If $z$ is a complex root, then $z^{2}$ is also a root, and so is $z^{2^{n}}$ $\forall n$. So, it must exist $p$ and $q$ such that $z^{2^{p}}=z^{2^{q}}$ and $z$ is either $0$, either $e^{\frac{2\pi}{n}i}$.
If $z$ is a complex root, $(z+1)^{2}$ is also a root, so $0$ can't be a root (since $1$ would be a root, and then $4, 25, ...$), and $(z+1)^{2}$ must also be some $e^{\frac{2\pi}{k}i}$,
But the only complex on the unit circle such that $(z+1)$ be also on the unit circle are $j=e^{\frac{2\pi}{3}i}$ and $j^{2}=e^{-\frac{2\pi}{3}i}$.
So the only possible roots are $j$ and $j^{2}$ and we must have $f(x)=a(x-j)^{p}(x-j^{2})^{q}$
Since we are looking for real polynomials, $p=q$ and we must have $f(x)=(x^{2}+x+1)^{p}$ or $f(x)=0$
And it is easy to check that this necessary condition works.
--
Patrick
\end{solution}
\begin{solution}[by \href{https://artofproblemsolving.com/community/user/5820}{N.T.TUAN}]
This is another idea http://www.mathlinks.ro/Forum/viewtopic.php?t=132186
\end{solution}
*******************************************************************************
-------------------------------------------------------------------------------
\begin{problem}[Posted by \href{https://artofproblemsolving.com/community/user/10512}{mathmanman}]
Do there exist a sequence $a_{1}, a_{2}, a_{3}, \ldots$ of real numbers and a non-constant polynomial $P(x)$ such that $a_{m}+a_{n}=P(mn)$ for every positive integral $m$ and $n?$
\begin{italicized}Proposed by A. Golovanov\end{italicized}
\flushright \href{https://artofproblemsolving.com/community/c6h151112}{(Link to AoPS)}
\end{problem}
\begin{solution}[by \href{https://artofproblemsolving.com/community/user/29428}{pco}]
\begin{tcolorbox}Do there exist a sequence $a_{1}, a_{2}, a_{3}, \ldots$ of real numbers and a non-constant polynomial $P(x)$ such that $a_{m}+a_{n}=P(mn)$ for every positive integral $m$ and $n?$\end{tcolorbox}
It does not :
$m=1$ $\Rightarrow $ $a_{n}=P(n)-a_{1}$
So we have $P(m)+P(n)-2a_{1}=P(mn)$ and for example $2P(n)-2a_{1}=P(n^{2})$ which can't be true $\forall n$ (take $n\rightarrow+\infty$) if $P(x)$ is non constant.
--
Patrick
\end{solution}
\begin{solution}[by \href{https://artofproblemsolving.com/community/user/16261}{Rust}]
\begin{tcolorbox}Do there exist a sequence $a_{1}, a_{2}, a_{3}, \ldots$ of real numbers and a non-constant polynomial $P(x)$ such that $a_{m}+a_{n}=P(mn)$ for every positive integral $m$ and $n?$\end{tcolorbox}
We get $a_{n}=P(n)-a_{1}, \ a_{1}=P(1)\/2.$ If $deg(P)=k$, then for big n $|a_{k}|n^{k}\/2=c_{1}n^{k}<|P(n)|<2a_{k}n^{k}=c_{2}n^{k}$.
Therefore $|P(n^{2})|>c_{1}n^{2k}>2c_{2}n^{k}>|P(n)|+|P(n)|$ - contradition.
\end{solution}
*******************************************************************************
-------------------------------------------------------------------------------
\begin{problem}[Posted by \href{https://artofproblemsolving.com/community/user/5820}{N.T.TUAN}]
Determine all polynomials $f (x)$ with real coeffcients that satisfy
\[f (x^{2}-2x) = f^{2}(x-2)\]
for all $x.$
\flushright \href{https://artofproblemsolving.com/community/c6h151180}{(Link to AoPS)}
\end{problem}
\begin{solution}[by \href{https://artofproblemsolving.com/community/user/29428}{pco}]
\begin{tcolorbox}Determine all polynomials $f (x)$ with real coeffcients that satisfy
\[f (x^{2}-2x) = f^{2}(x-2) \]
for all $x.$\end{tcolorbox}
Let $g(x)=f(x-1)$, then $g((x-1)^{2})=g^{2}(x-1)$ and $g(x^{2})=g^{2}(x)$
Then, if $z$ is a complex nonzero root of $g(x)$, then $z^{2}$ is also a root and, since we have a finite number of roots, all nonzero roots have their modulus equal to 1.
But then, if $e^{ix}$ $(x\in[0,2\pi))$ is a complex root, $e^{i\frac{x}{2}}$ is also a root, and so are $e^{i\frac{x}{2^{k}}}$ $\forall k\in\mathbb{N}$, and $x$ must be 0 (else we have an infinite number of roots). But, if $1=e^{0*i}$ is a root, so is $-1=e^{i\pi}$ and $e^{i\frac{\pi}{2^{k}}}$ $\forall k\in\mathbb{N}$.
So $g(x)$ can only have zeroes roots and we have $g(x)=ax^{n}$. This necessary condition works if $a^{2}=a$, so if $a=0$ or $a=1$
The solutions to the initial problem are :
$f(x)=0$
$f(x)=(x+1)^{n}$ $\forall n\in\mathbb{N}\cup\{0\}$
--
Patrick
\end{solution}
\begin{solution}[by \href{https://artofproblemsolving.com/community/user/5820}{N.T.TUAN}]
$g^{2}(x)=g(x^{2}).$(*)
If $g\equiv C$ then $C=0$ or $1$.
If $g\not\equiv C$ , write $g(x)=a_{n}x^{n}+a_{n-1}x^{n-1}+...+a_{0}(a_{n}\not = 0)$. By (*) we have $a_{n}x^{2n}=a_{n}^{2}x^{2n}$ or $a_{n}=1$. Now, we'll prove that $a_{0}=a_{1}=...=a_{n-1}=0$. Assume that there exist $i\in\{0,1,...,n-1\}$ such that $a_{i}\not = 0$, put $k=\max \{i|a_{i}\not = 0\}$, then by see at $x^{n+k}$ in (*) we have $2a_{k}=0$, contradiction!
\end{solution}
*******************************************************************************
-------------------------------------------------------------------------------
\begin{problem}[Posted by \href{https://artofproblemsolving.com/community/user/18420}{aviateurpilot}]
Suppose that
\[\prod_{n=1}^{1996}(1+nx^{3^{n}})=1+\sum_{i=1}^{m}a_{m}x^{k_{i}},\]
where $a_i$ (for $i=1,2,\ldots,m$) are non-zero real numbers and $k_{i}<k_{i+1}$ for all $i=1,2,\ldots,m-1$. Find $a_{1996}$.
\flushright \href{https://artofproblemsolving.com/community/c6h152574}{(Link to AoPS)}
\end{problem}
\begin{solution}[by \href{https://artofproblemsolving.com/community/user/29428}{pco}]
\begin{tcolorbox}$\prod_{n=1}^{1996}(1+nx^{3^{n}})=1+\sum_{i=1}^{m}a_{m}x^{k_{i}}$
and $\prod_{i=1}^{m}a_{i}\neq 0\ and\ \forall i\in\{1,2,..,m-1\},\ k_{i}<k_{i+1}$
find $a_{1996}$\end{tcolorbox}
Nice problem !
It's easy to see that the $k_{i}$ are all sum of $p$ different powers of $3$, with $1\leq p \leq 1996$
They are ordered in the same order that the numbers obtained by replacing $3$ by $2$ :
$1\rightarrow 2^{0}\rightarrow 3^{1}$
$2\rightarrow 2^{1}\rightarrow 3^{2}$
$3\rightarrow 2^{1}+2^{0}\rightarrow 3^{2}+3^{1}$
$4\rightarrow 2^{2}\rightarrow 3^{3}$
$5\rightarrow 2^{2}+2^{0}\rightarrow 3^{3}+3^{1}$
...
$1996\rightarrow 2^{10}+2^{9}+2^{8}+2^{7}+2^{6}+2^{3}+2^{2}\rightarrow 3^{11}+3^{10}+3^{9}+3^{8}+3^{7}+3^{4}+3^{3}$
And so $a_{1996}=11*10*9*8*7*4*3=665280$
\end{solution}
\begin{solution}[by \href{https://artofproblemsolving.com/community/user/18420}{aviateurpilot}]
good,
me also I found $\forall (a,b)\in ([2,+\infty[\times [10,+\infty[)\cap \mathbb{N}^{2},\ a_{1996}=665280$
if $\prod_{n=1}^{b}(1+nx^{a^{n}})=1+\sum_{i=1}^{m}a_{i}x^{k_{i}}$ where $a_{i}\neq 0,k_{i+1}>k_{i}$
\end{solution}
*******************************************************************************
-------------------------------------------------------------------------------
\begin{problem}[Posted by \href{https://artofproblemsolving.com/community/user/18653}{maky}]
The problem is about real polynomial functions, denoted by $f$, of degree $\deg f$.
a) Prove that a polynomial function $f$ can`t be wrriten as sum of at most $\deg f$ periodic functions.
b) Show that if a polynomial function of degree $1$ is written as sum of two periodic functions, then they are unbounded on every interval (thus, they are "wild").
c) Show that every polynomial function of degree $1$ can be written as sum of two periodic functions.
d) Show that every polynomial function $f$ can be written as sum of $\deg f+1$ periodic functions.
e) Give an example of a function that can`t be written as a finite sum of periodic functions.
\begin{italicized}Dan Schwarz\end{italicized}
\flushright \href{https://artofproblemsolving.com/community/c6h153374}{(Link to AoPS)}
\end{problem}
\begin{solution}[by \href{https://artofproblemsolving.com/community/user/18728}{edriv}]
a) By induction. Let $f$ be a polynomial that can be written as sum of $\deg f = n$ periodic functions $f_{1},f_{2},\ldots,f_{n}$ with periods $t_{1},t_{2},\ldots,t_{n}$.
We have:
$f(x) = f_{1}(x)+\ldots+f_{n}(x)$
Since $f_{1}(x)-f_{1}(x+t_{1}) = 0$, we get:
$f(x)-f(x+t_{1}) = f_{2}(x)-f_{2}(x+t_{1})+\ldots+f_{n}(x)-f_{n}(x+t_{1})$
Note that $f(x)-f(x+t_{1})$ is a polynomial of degree $n-1$, and it's written as sum of $n-1$ periodic functions.
So we can "go down" until a polynomial of degree 1 is written as sum of only one periodic function... but a polynomial of degree 1 is not periodic! :)
b) Since $f(x)$ is periodic if and only if $af(x)+b$ is, with a,b nonzero constants, and the same for boundedness, we can suppose the polynomial to be x.
Let $f_{1}(x)+f_{2}(x) = x$ for all x, and the period of $f_{1},f_{2}$ be $t_{1},t_{2}$.
Without loss of generality we suppose $f_{1}(0) = 0$. Then $f_{1}(nt_{1}) = 0$ for all integers n, and $f_{2}(nt_{1}) = kt_{1}$ for all integers n, and $f_{2}(nt_{1}+mt_{2}) = nt_{1}$ for all integers n,m. If the ratio $\frac{t_{1}}{t_{2}}$ is rational, than $f_{1}(x)+f_{2}(x)$ is a periodic function, but this is impossible. Therefore $\frac{t_{1}}{t_{2}}$ is irrational, and by a well-known theorem $nt_{1}+mt_{2}$ can go as near as we want to every real.
For each $\epsilon > 0$, there are integers n,m such that $0<nt_{1}+mt_{2}<\epsilon$, and we can also make $nt_{1}$ very large by going nearer to 0, and then multiplying n,m by a large factor (if $\epsilon < |t_{2}|$ we must have $n \ge 1$).
Then we can also go near to each real a with a very large $nt_{1}$, to which corresponds a very large value of $f_{2}$ which can't be bounded in an interval. The same for $f_{1}$.
\end{solution}
\begin{solution}[by \href{https://artofproblemsolving.com/community/user/18728}{edriv}]
c) I think that here we need the Axiom of Choice.
Take two nonzero reals $t_{1},t_{2}$ such that $\frac{t_{1}}{t_{2}}$ is irrational. Let $A = \{nt_{1}+mt_{2}| n,m \in \mathbb{Z}\}$. $a-b \in A$ is an equivalence relation among the reals. Let $[a]$ be the equivalence class of a, and let $c: \{[a]: a \in \mathbb{R}\}\rightarrow \mathbb{R}$ a choice function of the classes.
Using the definition of our equivalence relation and the fact that $\frac{t_{1}}{t_{2}}$ is irrational, we show that for each $x \in \mathbb{R}$ there exist unique integers n,m such that $x = c([x])+nt_{1}+mt_{2}$. Then we define:
$f_{1}(x) = f_{1}(c([x])+nt_{1}+mt_{2}) = c([x])+mt_{2}$
$f_{2}(x) = f_{2}(c([x])+nt_{1}+mt_{2}) = nt_{1}$
$f_{1},f_{2}$ are periodic, since obviously $f_{1}(x+t_{1})= f_{1}(c([x])+(n+1)t_{1}+mt_{2})= c([x])+mt_{2}= f_{1}(x)$ and the same for $f_{2}$.
And also $f_{1}(x)+f_{2}(x) = c([x])+mt_{2}+nt_{1}= x$.
d) This can be done by induction, using c). Suppose the statament is true for all polynomial with deg < n, and that you can choose freely the periods of the functions of which the polynomial is sum. (not completely freely, their ratio is irrational).
Take the polynomial f with $\deg f = n$. Choose a number $t_{n+1}$ as you want.
Consider $f(x)-f(x-t_{n+1})$. It is a polynomial of degree n-1. Then there exists periodic functions such that
$f(x)-f(x-t_{n+1})= g_{1}(x)+\ldots+g_{n}(x)$.
We first prove the lemma: if $g$ is a periodic function of period t, and $\frac{t}{t_{1}}$ is irrational, then there exists a function $g'$ such that $g(x) = g'(x)-g'(x-t_{1})$ and $g'$ is still periodic of period t.
The proof is somehow the same of the point c)... with the same terminology, define:
$g'(x) = g'(c([x])+nt+m t_{1}) = \sum_{i=0}^{m}g(c([x])+nt+it_{1})$ if $m \ge 0$, 0 is m = -1 and $\sum_{i=1}^{-m-1}g(c([x])+nt+-it)$ if $m<-1$.
Then it's easily seen that g' satisfies $g'(x)-g'(x-t_{1}) = g(x)$ and $g'(x) = g'(x+t)$.
Using the lemma, we get that there are periodic functions such that:
$f(x)-f(x-t_{n+1}) = f_{1}(x)-f_{1}(x-t_{n+1})+\ldots+f_{n}(x)-f_{n}(x-t_{n+1})$
Now we have just to add the suitable function $f_{n+1}$.
For each $0\le x < t_{n+1}$ define:
$f_{n+1}(x) = f(x)-f(x-t_{n+1})-f_{1}(x)+f_{1}(x-t_{n+1})+\ldots-f_{n}(x)+f_{n}(x-t_{n+1})$
And then extend $f_{n+1}$ to the reals using periodicity, and check that
$f(x) = f_{1}(x)+f_{2}(x)+\ldots+f_{n}(x)$ for all real x. :)
\end{solution}
\begin{solution}[by \href{https://artofproblemsolving.com/community/user/18653}{maky}]
my solution to c)
[hide="solution"]
let $B$ be a Hamel-basis of $\mathbb{R}$ over $\mathbb{Q}$ (as a vectorial space), and let $B_{1},B_{2}$ be a partition of it.
now, take functions $f_{1},f_{2}$, define them over $B$, then extend them as linear maps :
if $x\in B_{1}$, then $f_{1}(x)=x$ and $f_{2}(x)=0$
if $x\in B_{2}$, then $f_{1}(x)=0$ and $f_{2}(x)=x$.
now, for $y\in \mathbb{R}$, $y=\sum a_{i}x_{i}$ with $x_{i}\in B$ and $a_{i}\in \mathbb{Q}$.
$f_{1}(y)+f_{2}(y)=\sum_{x_{i}\in B_{1}}a_{i}f_{1}(x_{i})+\sum_{x_{i}\in B_{2}}a_{i}f_{2}(x_{i}) = \sum a_{i}x_{i}= y$.
also it is easy to check that $f_{1}$ is periodic (with period any $x\in B_{1}$) and $f_{2}$ is periodic also (with period any $x\in B_{2}$).
[\/hide]
edit : oops, i just said a biiiiiiiiiiiiiig stupid thing. :blush:
\end{solution}
\begin{solution}[by \href{https://artofproblemsolving.com/community/user/29428}{pco}]
\begin{tcolorbox}my solution to c) (without AC)
let $B$ be a Hamel-basis of $\mathbb{R}$ over $\mathbb{Q}$ (as a vectorial space), ...
\end{tcolorbox}
But this needs AC !
\end{solution}
\begin{solution}[by \href{https://artofproblemsolving.com/community/user/29428}{pco}]
\begin{tcolorbox} e) give an example of a function that can`t be written as a finite sum of periodic functions.\end{tcolorbox}
Take $f(x)=e^{x}$
Demo:
If $e^{x}$ is sum of $n$ periodic functions $e^{x}=\sum_{i=1}^{n}f_{i}(x)$, with $f_{i}(x+t_{i})=f(x)$ and $t_{i}>0$, then :
$e^{x+t_{n}}=\sum_{i=1}^{n-1}f_{i}(x+t_{n})+f_{n}(x+t_{n})=\sum_{i=1}^{n-1}f_{i}(x+t_{n})+f_{n}(x)$
$e^{x+t_{n}}-e^{x}= \sum_{i=1}^{n-1}(f_{i}(x+t_{n})-f_{i}(x))$
$e^{x}=\sum_{i=1}^{n-1}\frac{f_{i}(x+t_{n})-f_{i}(x)}{e^{t_{n}}-1}$
$e^{x}=\sum_{i=1}^{n-1}h_{i}(x)$, with $h_{i}(x)=\frac{f_{i}(x+t_{n})-f_{i}(x)}{e^{t_{n}}-1}$, $h_{i}(x+t_{i})=h_{i}(x)$ and $t_{i}>0$
So, if $e^{x}$ is sum of $n$ periodic functions, $e^{x}$ is sum of $n-1$ periodic functions, which implies $e^{x}$ is sum of $1$ periodic function, which is false.
So $e^{x}$ can't be sum of a finite number of periodic functions.
\end{solution}
\begin{solution}[by \href{https://artofproblemsolving.com/community/user/18728}{edriv}]
Nice!
Here is my proof:
Suppose a function $f$ is sum of n periodic functions $f_{1},\ldots,f_{n}$ with periods $t_{1},\ldots,t_{n}$, $t_{i}\neq 0$.
Let $T = \{t_{1},\ldots,t_{n}\}$. Then you can see by induction that, for all real x,
$\sum_{X \subset T}(-1)^{|X|}f\left( x+\sum_{t \in X}t \right) = 0$.
I think that, with some conditions on the $t_{i}$, this can become a sufficient condition.
If we put $f(x) = e^{x}$, we can factor the condition as:
$e^{x}(e^{t_{1}}-1)(e^{t_{2}}-1)\cdots (e^{t_{n}}-1) = 0$
But this is nonzero because all $t_{i}$ are different from 0 :D
\end{solution}
*******************************************************************************
-------------------------------------------------------------------------------
\begin{problem}[Posted by \href{https://artofproblemsolving.com/community/user/21482}{Yosh...}]
Let $ k$ be the smallest positive integer with the following property:
There are distinct integers $ m_{1},m_{2},m_{3},m_{4},m_{5}$ such that the polynomial \[p(x)=(x-m_{1})(x-m_{2})(x-m_{3})(x-m_{4})(x-m_{5})\] has exactly $k$ non-zero integers coefficients.
Find a set of integers $\{m_{1},m_{2},m_{3},m_{4},m_{5}\}$ for which this minimum $k$ is achieved
\flushright \href{https://artofproblemsolving.com/community/c6h155855}{(Link to AoPS)}
\end{problem}
\begin{solution}[by \href{https://artofproblemsolving.com/community/user/29428}{pco}]
\begin{tcolorbox}Let $k$ be the smallest positive integer with the following property:
There are distinct positive integers $m_{1},m_{2},m_{3},m_{4},m_{5}$
such that the polynomial $p(x)=(x-m_{1})(x-m_{2})(x-m_{3})(x-m_{4})(x-m_{5})$ has exactly $k$ non-zero integers coefficients.
Find a set of integers $m_{1},m_{2},m_{3},m_{4},m_{5}$ for which this minimum $k$ is achieved\end{tcolorbox}
I don't understand : if all $m_{i}>0$, none of the coefficients of $p(x)=(x-m_{1})(x-m_{2})(x-m_{3})(x-m_{4})(x-m_{5})$ can be zero : coeff of $x^{5},x^{3},x$ are all $>0$ and coeff of $x^{4},x^{2},1$ are all $<0$.
So $k=6$ and any polynomial verify the property.
\end{solution}
\begin{solution}[by \href{https://artofproblemsolving.com/community/user/21482}{Yosh...}]
Thx for the correction..
\end{solution}
\begin{solution}[by \href{https://artofproblemsolving.com/community/user/29428}{pco}]
\begin{tcolorbox}Let $ k$ be the smallest positive integer with the following property:
There are distinct integers $ m_{1},m_{2},m_{3},m_{4},m_{5}$
such that the polynomial $ p(x)=(x-m_{1})(x-m_{2})(x-m_{3})(x-m_{4})(x-m_{5})$ has exactly $ k$ non-zero integers coefficients.
Find a set of integers $ m_{1},m_{2},m_{3},m_{4},m_{5}$ for which this minimum $ k$ is achieved\end{tcolorbox}
Hello !
With this correction, I claim $ k=3$.
1) $ k=0$ is impossible : it would mean $ p(x)=0$
2) $ k=1$ is impossible : it would mean $ p(x)=x^{5}$ and all $ m_{i}$ would be equal (to 0)
3) $ k=2$ is impossible : it would mean $ p(x)=x^{5}+ax^{4}$ or $ p(x)=x^{5}+ax^{3}$ or $ p(x)=x^{5}+ax^{2}$ or $ p(x)=x^{5}+ax$ or $ p(x)=x^{5}+a$ :
$ p(x)=x^{5}+ax^{4}$ implies four identical roots 0, which is impossible
$ p(x)=x^{5}+ax^{3}$ implies three identical roots 0, which is impossible
$ p(x)=x^{5}+ax^{2}$ implies two identical roots 0, which is impossible
$ p(x)=x^{5}+ax$ implies a root 0 and the four roots of $ x^{4}=-a$. But this equation can't have four real roots, so impossible
$ p(x)=x^{5}+a$ implies $ m_{i}$ are the five roots of $ x^{5}+a=0$. But this equation can't have five real roots, so impossible
3) $ k=3$ is possible :
Take $ m_{1}=0,m_{2}=a,m_{3}=-a,m_{4}=b,m_{5}=-b$ with $ a\neq b$ and $ ab\neq 0$. Then :
$ p(x)=x^{5}-(a^{2}+b^{2})x^{3}+a^{2}b^{2}x$ and $ p(x)$ has exactly 3 non-zero integer coefficients.
\end{solution}
*******************************************************************************
-------------------------------------------------------------------------------
\begin{problem}[Posted by \href{https://artofproblemsolving.com/community/user/5820}{N.T.TUAN}]
Find all $ P\in\mathbb{R}[x]$ such that \[1+P(x)=\frac{P(x-1)+P(x+1)}{2}\] holds for all $x\in\mathbb R.$
\flushright \href{https://artofproblemsolving.com/community/c6h159183}{(Link to AoPS)}
\end{problem}
\begin{solution}[by \href{https://artofproblemsolving.com/community/user/29428}{pco}]
\begin{tcolorbox}Find all $ P\in\mathbb{R}[x]$ such that $ 1+P(x)=\frac{P(x-1)+P(x+1)}{2}\forall x\in\mathbb R.$\end{tcolorbox}
If $ P(x)$ verifies the requested condition, then all the derivatives of $ P(x)$ verify $ P^{(i)}(x)=\frac{P^{(i)}(x-1)+P^{(i)}(x+1)}{2}$
It is easy to see that no quadratic polynomial $ Q(x)=ax^{2}+bx+c$ with $ a\neq 0$ verify this last equation $ Q(x)=\frac{Q(x-1)+Q(x+1)}{2}$ since $ Q(0)=c\neq a+c=\frac{Q(-1)+Q(1)}{2}$
So $ deg(P)\leq 2$ and $ P(x)=ax^{2}+bx+c$. Putting back this expression in the original equation, we find $ a=1$
And all the solutions are polynomials $ P(x)=x^{2}+bx+c$
\end{solution}
\begin{solution}[by \href{https://artofproblemsolving.com/community/user/29386}{mszew}]
[hide="Is it a hint?"]Transforming the equation to $ P(x+1)=2P(x)+2-P(x-1)$ and using a kind of binet formula like fibonacci sequence[\/hide]
\end{solution}
\begin{solution}[by \href{https://artofproblemsolving.com/community/user/18420}{aviateurpilot}]
\begin{tcolorbox}Find all $ P\in\mathbb{R}[x]$ such that $ 1+P(x)=\frac{P(x-1)+P(x+1)}{2}\forall x\in\mathbb R.$\end{tcolorbox}
we take $ f(x)=p(x)-x^{2}$
we have $ f(x)=\frac{f(x-1)+f(x+1)}{2}$ then $ f(x)=ax+b$
so $ p(x)=f(x)+x^{2}=x^{2}+ax+b$
\end{solution}
*******************************************************************************
-------------------------------------------------------------------------------
\begin{problem}[Posted by \href{https://artofproblemsolving.com/community/user/22328}{sinajackson}]
Find all polynomials
\[P_{n}(x)=n!x^{n}+a_{n-1}x^{n-1}+\cdots+a_{1}x+(-1)^{n}n(n+1)\]
with integer coefficients such that $P_n$ has $ n$ real roots $ x_{1},x_{2},\ldots,x_{n}$ that have the following condition: for all $ k=1,2,\ldots,n$, we have $ k\leq x_{k}\leq k+1$.
\flushright \href{https://artofproblemsolving.com/community/c6h162115}{(Link to AoPS)}
\end{problem}
\begin{solution}[by \href{https://artofproblemsolving.com/community/user/16261}{Rust}]
$ x_{1}...x_{n}=\frac{n+1}{(n-1)!}<1$ if $ n\ge 3$. If n=2, then $ P_{2}(x)=2x^{2}+a_{1}x+6=2(x-1)(x-3),a_{1}=-8$.
Therefore it may be if and only if $ n\le 2$.
\end{solution}
\begin{solution}[by \href{https://artofproblemsolving.com/community/user/29428}{pco}]
\begin{tcolorbox}$ x_{1}...x_{n}=\frac{n+1}{(n-1)!}<1$ if $ n\ge 3$. If n=2, then $ P_{2}(x)=2x^{2}+a_{1}x+6=2(x-1)(x-3),a_{1}=-8$.
Therefore it may be if and only if $ n\le 2$.\end{tcolorbox}
Two little mistakes, Rust :
$ 1).$ For $ n=3$, $ \frac{n+1}{(n-1)!}=2>1$. In fact, we must have $ n!\leq x_{1}...x_{n}\leq (n+1)!$ and this\end{underlined} is not verified for n=3.
$ 2).$ For $ n=2$, two values for $ a_{1}$ fit : $ a_{1}=-7$ and $ a_{1}=-8$ (remember roots need not to be integers).
So exactly three polynomials are solutions :
$ 2x^{2}-7x+6$
$ 2x^{2}-8x+6$
$ x-2$
\end{solution}
*******************************************************************************
-------------------------------------------------------------------------------
\begin{problem}[Posted by \href{https://artofproblemsolving.com/community/user/32514}{TTsphn}]
Suppose that $ P(x)$ and $Q(x)$ are polynomials with real coefficients such that for all $x$, $P(x)$ is an integer if and only if $Q(x)$ is an integer. Prove that $P(x)-Q(x)$ is a constant.
\flushright \href{https://artofproblemsolving.com/community/c6h166088}{(Link to AoPS)}
\end{problem}
\begin{solution}[by \href{https://artofproblemsolving.com/community/user/32541}{gregg}]
$ P(x)\in Z\Leftrightarrow P(x)=a\forall x\in R[x]$
$ Q(x)\in Z\Leftrightarrow Q(x)=b\forall x\in R[x]$
So $ P(x)-Q(x) = a-b$
We put a-b=c and here is the answer
I am not sure though about the answer.
\end{solution}
\begin{solution}[by \href{https://artofproblemsolving.com/community/user/18418}{\u0391\u03c1\u03c7\u03b9\u03bc\u03ae\u03b4\u03b7\u03c2 6}]
No :| .Suppose that P(x)=x^2+1 & Q(x)=x then P(x)-Q(x)=x^2-x-1......
\end{solution}
\begin{solution}[by \href{https://artofproblemsolving.com/community/user/32514}{TTsphn}]
This problem haven't solved .
It is very good.
Has some problem from this problem
Find all $ P(x)\in R[x]$ such that
$ P(x+1)\in Z\leftrightarrow P(x)\in Z$
\end{solution}
\begin{solution}[by \href{https://artofproblemsolving.com/community/user/29428}{pco}]
\begin{tcolorbox}Suppose $ P(x),Q(x)\in R[x]$
$ \forall x : P(x)\in Z\Leftrightarrow Q(x)\in Z$
Prove that $ P(x)-Q(x) = c$\end{tcolorbox}
I think we miss some precision :
Let $ P(x)=x$ and $ Q(x)=1-x$, we have $ \forall x : P(x)\in Z\Leftrightarrow Q(x)\in Z$ but $ P(x)-Q(x) = 2x-1\neq c$
maybe you wanted to add "monic" polynomials ?
\end{solution}
\begin{solution}[by \href{https://artofproblemsolving.com/community/user/32514}{TTsphn}]
I am sorry $ P(x)+Q(x)$ or $ P(x)-Q(x)$=c
\end{solution}
\begin{solution}[by \href{https://artofproblemsolving.com/community/user/29428}{pco}]
\begin{tcolorbox}Suppose $ P(x),Q(x)\in R[x]$
$ \forall x : P(x)\in Z\Leftrightarrow Q(x)\in Z$
Prove that $ P(x)-Q(x) = c$\end{tcolorbox}
Here is a solution :